नमूने के अधिकतम भाग का विचलन क्या है?


13

मैं यादृच्छिक चर के एक सेट के अधिकतम के संस्करण पर सीमा की तलाश कर रहा हूं। दूसरे शब्दों में, मैं लिए बंद फॉर्मूले की तलाश कर रहा हूं , जैसे कि जहां X = \ {X_1, \ ldots, X_n \} एक निश्चित है परिमित के साथ M यादृच्छिक चर का सेट का अर्थ है \ mu_1, \ ldots, \ mu_M और variances \ sigma_1 ^ 2, \ ldots, \ sigma_M ^ 2B

Var(maxiXi)B,
X={X1,,XM}Mμ1,,μMσ12,,σM2

मैं उस

Var(maxiXi)iσi2,
लेकिन यह बाउंड बहुत ढीला लगता है। एक संख्यात्मक परीक्षण से संकेत मिलता है कि B=maxiσi2 एक संभावना हो सकती है, लेकिन मैं यह साबित नहीं कर पाया हूं। किसी भी मदद की सराहना की है।

3
(क्या आप मान लेना चाहते हैं कि Xi स्वतंत्र हैं?) अनुमान प्रशंसनीय है लेकिन गलत प्रतीत होता है। उदाहरण के लिए, कुछ परीक्षण करें जहाँ Xi CDF 1x1s , 1x , s \ gt 3 के साथ iid हो s>3। उनके सामान्य विचरण के सापेक्ष, उनके अधिकतम का विचरण, M ग्रो के बिना बाउंड के रूप में बढ़ता है।
whuber

@whuber धन्यवाद, यही कारण है कि मैं यह साबित करने में सक्षम नहीं था कि :) मैं वास्तव में उस मामले में दिलचस्पी रखता हूं जहां Xi स्वतंत्र हैं। बस स्पष्ट करने के लिए, मैं ज्यादातर सामान्य सीमाओं में दिलचस्पी रखता हूं जो केवल पहले दो क्षणों का उपयोग करते हैं। मुझे यकीन नहीं है कि सामान्य संस्करण की तुलना में शार्पर जनरल सीमाएं भी मौजूद हैं या नहीं।
पीटर

1
मुझे यह इंगित करना चाहिए कि आपकी राशि बाध्य है (यह मानते हुए कि यह सही है - प्रमाण का एक स्केच देखना अच्छा होगा) तंग है। उदाहरण के लिए, को अंतराल पर समर्थित किया जाना चाहिए जिसमें variances से अधिक न हो और को पर समर्थित होने दें । तब जैसा कि, विचरण , लेकिन असमानता को उतना ही कड़ा किया जा सकता है जितना आप सिकुड़ते । X2,,XM[,a]ε2X1[a,]maxiXi=X1σ12σ12+(M1)ε2ε2
whuber

1
Iid डेटा के लिए, चरम मूल्य सिद्धांत वितरण के वर्गों को प्रदान करता है, जिसमें नमूना अधिकतम रूपांतरित होता है, मूल वितरण की पूंछ पर कुछ शर्तों के साथ विभिन्न वर्गों के स्पर्शोन्मुख वितरण देता है। इसलिए मुझे संदेह है कि आप केवल दो क्षणों के आधार पर एक अच्छी बाध्यता प्राप्त कर पाएंगे, हालांकि मैं केवल सिद्धांत से परिचित हूं।
StasK

जवाबों:


9

किसी भी के लिए यादृच्छिक परिवर्तनीय , सबसे अच्छा सामान्य बाध्य है के रूप में मूल प्रश्न में कहा गया है। यहाँ एक प्रूफ स्केच है: यदि X, Y IID हैं तो । संभवतः आश्रित चर के एक वेक्टर को देखते हुए , let एक ही संयुक्त वितरण के साथ एक स्वतंत्र वेक्टर हो। किसी भी , हमें संघ द्वारा बाध्य किया जाता है कि , और इस एकीकृत से करने के लिए पैदावार दावा किया असमानता।nXiVar(maxXi)iVar(Xi)E[(XY)2]=2Var(X)(X1,,Xn)(Y1,,Yn)r>0P[|maxiXimaxiYi|2>r]iP[|XiYi|2>r]dr0

यदि प्रायिकता की घटनाओं के IID संकेतक हैं , तो प्रायिकता घटना का सूचक है । फिक्सिंग और को शून्य करने देते हैं, हम और ।XiϵmaxXinϵ+O(n2ϵ2)nϵVar(Xi)=ϵϵ2Var(maxiXi)=nϵ+O(n2ϵ2)


3

MathOverflow पर एक प्रश्न इस प्रश्न से संबंधित है।

IID यादृच्छिक चर के लिए, वें उच्चतम को एक क्रम सांख्यिकीय कहा जाता है ।k

यहां तक ​​कि IID बर्नौली यादृच्छिक चर के लिए, माध्यिका के अलावा किसी भी क्रम सांख्यिकीय का संस्करण जनसंख्या के विचरण से अधिक हो सकता है। उदाहरण के लिए, यदि है संभावना के साथ और संभावना के साथ और है, तो अधिकतम है संभावना के साथ , तो जनसंख्या का विचरण है , जबकि विचरण अधिकतम ।Xi11/1009/10M=10111/e0.090.23

आदेश के आँकड़ों के दो पेपर यहाँ दिए गए हैं:

यांग, एच। (1982) "माध्यिका और कुछ अन्य आदेश आँकड़ों के संस्करण पर।" सांड। Inst। गणित। Acad। सिनिका, 10 (2) पीपी 197-204

पापदातोस, एन। (1995) "ऑर्डर आंकड़ों का अधिकतम विचरण।" एन। Inst। सांख्यिकीविद। गणित, 47 (1) पीपी 185-193

मेरा मानना ​​है कि दूसरे पेपर में अधिकतम के विचरण पर ऊपरी सीमा । वे बताते हैं कि समानता नहीं हो सकती है, लेकिन IID बर्नौली यादृच्छिक चर के लिए कोई भी कम मूल्य हो सकता है।Mσ2

हमारी साइट का प्रयोग करके, आप स्वीकार करते हैं कि आपने हमारी Cookie Policy और निजता नीति को पढ़ और समझा लिया है।
Licensed under cc by-sa 3.0 with attribution required.